Difference between revisions of "1966 AHSME Problems/Problem 24"

(Created page with "== Problem == If <math>Log_M{N}=Log_N{M},M \ne N,MN>0,M \ne 1, N \ne 1</math>, then <math>MN</math> equals: <math>\text{(A) } \frac{1}{2} \quad \text{(B) } 1 \quad \text{(C) } 2...")
 
(Solution)
Line 5: Line 5:
  
 
== Solution ==
 
== Solution ==
 
+
<math>\fbox{B}</math>
  
 
== See also ==
 
== See also ==

Revision as of 02:32, 15 September 2014

Problem

If $Log_M{N}=Log_N{M},M \ne N,MN>0,M \ne 1, N \ne 1$, then $MN$ equals:

$\text{(A) } \frac{1}{2} \quad \text{(B) } 1 \quad \text{(C) } 2 \quad \text{(D) } 10 \\ \text{(E) a number greater than 2 and less than 10}$

Solution

$\fbox{B}$

See also

1966 AHSME (ProblemsAnswer KeyResources)
Preceded by
Problem 23
Followed by
Problem 25
1 2 3 4 5 6 7 8 9 10 11 12 13 14 15 16 17 18 19 20 21 22 23 24 25 26 27 28 29 30
All AHSME Problems and Solutions

The problems on this page are copyrighted by the Mathematical Association of America's American Mathematics Competitions. AMC logo.png